O que é uma explicação intuitiva do seguinte fato matemático: [matemática] \ displaystyle \ sum\_ {n = 1} ^ {\ infty} {\ frac {1} {n ^ 2}} = \ frac {\ pi ^ 2} {6} [/ math]?

Melhor resposta

\ dfrac {1} {1 ^ 2} + \ dfrac {1} {2 ^ 2} + \ dfrac {1} {3 ^ 2} + \ dfrac {1} {4 ^ 2} + \ dfrac {1} {5 ^ 2} + … = \ dfrac {\ pi ^ 2} {6} \ tag {1}

Há uma prova excelente para isso, e essa prova é realmente como Euler provou essa identidade pela primeira vez. Claro, devo creditar meu professor por mostrar essa identidade para mim. (Todas as contas do Quora estão listadas na guia “Citações” no final desta resposta) Por último, o único cálculo necessário para entender esta prova é a regra de poder, que você ainda pode fazer sem saber.

\ dfrac {\ mathrm d} {\ mathrm dx} [x ^ n] = nx ^ {n-1} \ tag * {}

Começamos com um pouco de história da matemática. A identidade real de Euler é não e ^ {i \ pi} + 1 = 0. Na verdade, um matemático chamado Roger Cotes escreveu sobre isso décadas antes de Euler, mas Euler era mais famoso, então ele recebeu o crédito por sua descoberta. A identidade que provou ser a fama de Euler foi, na verdade,

\ dfrac {1} {1 ^ 2} + \ dfrac {1} {2 ^ 2} + \ dfrac {1} {3 ^ 2} … = \ dfrac {\ pi ^ 2} {6} \ tag * {}

* Curiosamente, existe uma lei, chamada Lei de Stigler, que afirma que nenhuma descoberta científica tem o nome de seu descobridor original. Claro, para consistência, esta lei foi proposta pela primeira vez por Robert Merton. Exemplos desta lei incluem a Identidade de Euler, descoberta por Roger Cotes, a Lei de Hubble, derivada de George Lemaitre e o Teorema de Pitágoras descoberto por matemáticos babilônios muito antes de Pitágoras. De qualquer forma, voltando à resposta.

Esse problema existia muito antes de Euler, mas não foi resolvido até ele. Matemáticos da época como Jacob Bernoulli, Johan Bernoulli, Leibniz e John Wallis já haviam trabalhado no problema muito antes de Euler, mas não conseguiram encontrar um valor exato para o problema em questão. Na verdade, esse problema começou a se tornar tão grande que ganhou seu próprio nome: o Problema de Basel.

Para provar que a soma de Euler converge em primeiro lugar, temos que reescrevê-la a partir deste

\ dfrac {1} {1 ^ 2} + \ dfrac {1} {2 ^ 2} + \ dfrac {1} {3 ^ 2} + \ dfrac {1} {4 ^ 2} … \ tag {1}

para isso.

\ dfrac {1} {1} \ cdot \ dfrac {1} {1} + \ dfrac 1 2 \ cdot \ dfrac 1 2+ \ dfrac 1 3 \ cdot \ dfrac 1 3+ \ dfrac 1 4 \ cdot \ dfrac 1 4 … \ tag {2}

Ambos são obviamente as mesmas coisas, mas apenas expressos de forma diferente. Vou mostrar por que (2) é mais útil do que (1) aqui em um segundo.

Pegue (2) e altere-o. Como é muito difícil expressar em palavras, terei apenas que mostrar a vocês:

\ dfrac {1} {1} \ cdot \ dfrac {1} {1} + \ dfrac 1 2 \ cdot \ dfrac 1 2+ \ dfrac 1 3 \ cdot \ dfrac 1 3+ \ dfrac 1 4 \ cdot \ dfrac 1 4 … \ tag {2}

\ implica

\ dfrac {1} {1} + \ dfrac 1 1 \ cdot \ dfrac 1 2+ \ dfrac 1 2 \ cdot \ dfrac 1 3+ \ dfrac 1 3 \ cdot \ dfrac 1 4 … \ tag {3}

Portanto, alteramos o valor da soma infinita, não se preocupe, não estou tentando passar por você. Vamos analisar (3).

Esta nova série parece ser maior que (2). O primeiro termo em (2) e (3) são obviamente iguais entre si. O segundo termo em (3) é certamente maior que (2) e vemos que esse processo continua até o infinito. Isso significa que se esta série (3) convergir, então a outra (2) também.

Portanto, esta parte pode não ser óbvia para a maioria das pessoas, o que é bom; também não foi óbvio para mim no início.

\ dfrac {1} {1} + \ dfrac 1 1 \ cdot \ dfrac 1 2+ \ dfrac 1 2 \ cdot \ dfrac 1 3+ \ dfrac 1 3 \ cdot \ dfrac 1 4 … \ tag {3}

Vamos cortar a série após os 4 primeiros termos e encontrar a soma parcial.

\ dfrac {1} {1} + \ dfrac 1 1 \ cdot \ dfrac 1 2+ \ dfrac 1 2 \ cdot \ dfrac 1 3+ \ dfrac 1 3 \ cdot \ dfrac 1 4 \ tag * {}

Para quem não sabe, esta série pode ser reescrita como:

\ dfrac {1} {1} + \ left (\ dfrac 1 1- \ dfrac 1 2 \ right) + \ left (\ dfrac 1 2 – \ dfrac 1 3 \ right) + \ left (\ dfrac 1 3- \ dfrac 1 4 \ right) \ tag * {}

Editar:

Recebi uma pergunta perguntando como podemos chegar à nova série, e esta foi minha resposta:

* Se você souber como chegar a essa etapa, então você pode pular esta próxima seção citada.

Sim, você pode chegar a essa etapa por expansão de fração parcial. A série em sua forma atual é esta:

1+ \ displaystyle \ sum\_ {n = 1} ^ {+ \ infty} \ dfrac {1} {n (n + 1)} \ tag * { }

Você pode assumir que o interior da soma pode ser representado por uma função de números reais, ou uma função de x, expandida em duas novas frações,

\ dfrac {1 } {x (x + 1)} = \ dfrac {A} {x} + \ dfrac {B} {x + 1} \ tag * {}

Multiplicando por um denominador comum,

1 = A (x + 1) + B (x) \ tag * {}

Já que x \ in \ mathbb R, vamos deixar x = 0 e encontrar A = 1 .Da mesma forma, deixando x = -1 nos dá B = -1, então podemos reescrever o argumento da soma como

1+ \ displaystyle \ sum\_ {n = 1} ^ {+ \ infty} \ left (\ dfrac {1} {n} – \ dfrac {1} {n + 1} \ right) \ tag * {}

A adição é associativa, então reescreva esta soma parcial:

\ left (\ dfrac 1 1+ \ dfrac 1 1 \ right) + \ left (- \ dfrac 1 2+ \ dfrac 1 2 \ right) + \ left (- \ dfrac 1 3+ \ dfrac 1 3 \ right ) – \ dfrac 1 4 \ tag * {}

Que então se torna trivial.

2- \ dfrac 1 4 \ tag * {}

Nós Agora vamos voltar para nossa série infinita (3) e substituir os primeiros 4 termos por 2- \ frac 1 4 e ver o que acontece a partir daí.

2- \ dfrac 1 4+ \ dfrac 1 4 \ cdot \ dfrac 1 5+ \ dfrac 1 5 \ cdot \ dfrac 1 6+ \ dfrac 1 6 \ cdot \ dfrac 1 7 … \ tag * {}

Usando o truque que usamos anteriormente,

2- \ dfrac 1 4+ \ dfrac 1 4- \ dfrac 1 5+ \ dfrac 1 5- \ dfrac 1 6+ \ dfrac 1 6- \ dfrac 1 7 … \ tag * {}

E agora o valor para esta soma infinita se torna aparente.

\ dfrac {1} {1} + \ dfrac 1 1 \ cdot \ dfrac 1 2+ \ dfrac 1 2 \ cdot \ dfrac 1 3+ \ dfrac 1 3 \ cdot \ dfrac 1 4 … = 2 \ tag * {}

Ótimo! Portanto, agora sabemos que a soma em questão converge para um valor menor que 2. Para os curiosos, \ frac {\ pi ^ 2} {6} \ approx 1.644.

Podemos agora começar a provar Identidade real de Euler:

Suponha que \ sin x possa ser expresso como algum polinômio infinito.

\ sin x = a + bx + cx ^ 2 + dx ^ 3 + … \ tag * {}

Agora podemos encontrar todos os termos do polinômio trivialmente. Comece deixando x = 0

\ sin 0 = a + b0 + c0 ^ 2 + d0 ^ 3 … \ tag * {}

0 = a \ tag * { }

Portanto, nosso novo polinômio infinito torna-se

\ sin x = bx + cx ^ 2 + dx ^ 3 + … \ tag * {}

Diferenciando ambos os lados

\ cos x = b + 2cx + 3dx ^ 2 … \ tag * {}

Definição x = 0,

1 = b \ tag * {}

Diferenciar e definir x = 0 nos dá um polinômio infinito para \ sin x. Se você continuar fazendo isso para sempre, acabará chegando à conclusão de que

\ sin x = 0 + 1x- \ dfrac {1} {2 \ cdot 3} x ^ 3 + … \ tag * {}

O que simplifica para

\ sin x = \ dfrac {x} {1!} – \ dfrac {x ^ 3} {3!} + \ dfrac {x ^ 5} {5!} – \ dfrac {x ^ 7} {7!} … \ tag * {}

Então, acabamos de recuperar a série Maclaurin para \ sin x. Peço desculpas, mas senti a necessidade de incluir a prova para isso, pois já estamos provando outras coisas de qualquer maneira.

Embora esta seja certamente uma expansão viável para \ sin x, Euler adotou uma abordagem diferente. Dê uma olhada no gráfico de \ sin x, \, x \ in [- \ pi, \ pi]. Sabemos que existem zeros em x = – \ pi, \, 0, \, \ pi, então se formos modelar este gráfico, podemos escrever uma função cúbica com zeros em – \ pi, \, 0, \, \ pi.

f (x) = x (\ pi + x) (\ pi-x) \ tag * {}

Que se parece com isto:

Claro, isso não se parece muito com f (x) = \ sin x em tudo, mas podemos escalá-lo multiplicando o função por alguma constante. Depois de muito mexer, vemos que a constante que faz o gráfico se ajustar melhor a \ sin x é \ frac {1} {\ pi ^ 2}. Vamos ver nosso novo gráfico de

f (x) = \ dfrac {1} {\ pi ^ 2} x (\ pi + x) (\ pi-x) \ tag * {}

O que é, embora não seja exato, muito melhor. Vamos manipular nossa função aqui e você verá por que isso acontece mais tarde.

f (x) = \ dfrac {1} {\ pi ^ 2} x (\ pi + x) (\ pi -x) \ tag * {}

f (x) = \ dfrac {1} {\ pi} (\ pi + x) \ cdot \ dfrac {1} {\ pi} (\ pi- x) x \ tag * {}

f (x) = x \ left (1+ \ dfrac {x} {\ pi} \ right) \ left (1- \ dfrac {x} {\ pi} \ right) \ tag * {}

f (x) = x \ left (1- \ dfrac {x ^ 2} {\ pi ^ 2} \ right) \ tag * {}

Mas não aproximamos a função inteira. Para fazer isso, precisaremos determinar novos termos que adicionam novos zeros em x = 2 \ pi, \, – 2 \ pi. Não vou mostrar a álgebra novamente e você está livre para verificá-la se quiser, mas nossa nova função se torna:

f (x) = x \ left (1- \ dfrac {x ^ 2} {\ pi ^ 2} \ right) \ left (1- \ dfrac {x ^ 2} {(2 \ pi) ^ 2} \ right) \ tag * {}

E a seguir padrão de adicionar novos termos para receber nossos novos zeros, nossa nova função modela perfeitamente o de \ sin x.

f (x) = \ sin x = x \ left (1- \ dfrac {x ^ 2} {\ pi ^ 2} \ right) \ left (1- \ dfrac {x ^ 2} {(2 \ pi) ^ 2} \ right) \ left (1- \ dfrac {x ^ 2} {(3 \ pi) ^ 2} \ right) \ left (1- \ dfrac {x ^ 2} {(4 \ pi) ^ 2} \ right) … \ tag {4}

E aqui são os gráficos, lado a lado.

* Embora não sejam exatos, este é o gráfico escrito em 7 termos . Peço desculpas, não pude sair até o infinito, mas não tive a noite toda. Isso será suficiente, no entanto, já que seu objetivo era mostrar as semelhanças entre este gráfico e \ sin x.

Estamos chegando lá, então preparem-se! Se desejar, clique fora desta resposta e veja se você pode fazer o resto do caminho a partir daqui. Boa sorte se você fizer isso!

Nós vamos nos sujeitar a pura tortura, então multiplique (4). Vou pular a álgebra porque não estamos aqui para enlouquecer.

f (x) = x \ left (1- \ dfrac {x ^ 2} {\ pi ^ 2} \ right ) \ left (1- \ dfrac {x ^ 2} {(2 \ pi) ^ 2} \ right) \ left (1- \ dfrac {x ^ 2} {(3 \ pi) ^ 2} \ right) \ left (1- \ dfrac {x ^ 2} {(4 \ pi) ^ 2} \ right) … \ tag {4}

f (x) = \ left (x- \ dfrac {x ^ 3} {\ pi ^ 2} \ right) \ left (1- \ dfrac {x ^ 2} {(2 \ pi) ^ 2} \ right) … \ tag * {}

f (x) = \ left (x + \ left (- \ dfrac {1} {\ pi ^ 2} – \ dfrac {1} {(2 \ pi) ^ 2} \ right) x ^ 3 + \ dfrac {1} {\ pi ^ 2} \ cdot \ dfrac {1} {(2 \ pi) ^ 2} x ^ 5 \ right) \ left (1- \ dfrac {x ^ 2} {(3 \ pi) ^ 2} \ right) …\ tag * {}

Vamos nos concentrar exclusivamente no coeficiente do termo x ^ 3 a partir daqui, então vamos encaixotá-lo.

f (x) = \ left (x + \ boxed {\ left (- \ dfrac {1} {\ pi ^ 2} – \ dfrac {1} {(2 \ pi) ^ 2} \ right) x ^ 3} + \ dfrac {1} {\ pi ^ 2} \ cdot \ dfrac {1} {(2 \ pi) ^ 2} x ^ 5 \ right) \ left (1- \ dfrac {x ^ 2} {(3 \ pi) ^ 2} … \ right) \ tag * {}

Ok, vamos agora multiplicar o próximo termo pelo primeiro. Mais uma vez, vou poupá-lo dos bits de álgebra.

x + \ boxed {\ left (- \ dfrac {1} {\ pi ^ 2} – \ dfrac {1} {(2 \ pi) ^ 2} – \ dfrac {1} {(3 \ pi) ^ 2} \ direita) x ^ 3} + (…) x ^ 5 + (…) x ^ 7 \ tag * {}

Então, a partir daqui, está bem claro como será o coeficiente em x ^ 3. Não precisamos mais fazer álgebra, pois podemos apenas seguir o padrão e assumir que isso continuará a acontecer para todos os termos. Depois disso, compararemos esta soma infinita com nossa série Maclaurin para \ sin x.

\ sin x = x + \ boxed {\ left (- \ dfrac {1} {\ pi ^ 2} – \ dfrac {1} {(2 \ pi) ^ 2} – \ dfrac {1} {(3 \ pi) ^ 2} – \ dfrac {1} {(4 \ pi) ^ 2} – \ enspace … \ enspace \ right) x ^ 3} + (…) x ^ 5 + (…) x ^ 7 \ tag * {}

\ sin x = \ dfrac {x} {1 !} – \ dfrac {x ^ 3} {3!} + \ dfrac {x ^ 5} {5!} – \ dfrac {x ^ 7} {7!} + \ dfrac {x ^ 9} {9!} – \ dfrac {x ^ {11}} {11!} + … \ tag * {}

Por comparação de coeficientes, vemos que

– \ dfrac {x ^ 3} {3!} = \ Left (- \ dfrac {1} {\ pi ^ 2} – \ dfrac {1} {(2 \ pi) ^ 2} – \ dfrac {1} {(3 \ pi) ^ 2} – \, … \ right) x ^ 3 \ tag * {}

Remova o x ^ 3 de ambos os lados.

– \ dfrac {1} {3 !} = – \ dfrac {1} {\ pi ^ 2} – \ dfrac {1} {(2 \ pi) ^ 2} – \ dfrac {1} {(3 \ pi) ^ 2} – \, .. . \ tag * {}

Multiplique ambos os lados por -1:

\ dfrac {1} {3!} = \ dfrac {1} {\ pi ^ 2} + \ dfrac {1} {(2 \ pi) ^ 2} + \ dfrac {1} {(3 \ pi) ^ 2} + \ dfrac {1} {(4 \ pi) ^ 2} + \ dfrac {1} { (5 \ pi) ^ 2} + … \ tag * {}

Multiplique por \ pi ^ 2

\ dfrac {\ pi ^ 2} {3!} = \ dfrac {1} {1 ^ 2} + \ dfrac {1} {2 ^ 2} + \ dfrac {1} {3 ^ 2} + \ dfrac {1} {4 ^ 2} + \ dfrac {1} { 5 ^ 2} + \ dfrac {1} {6 ^ 2} … \ tag * {}

Avalie 3! e aí está:

\ dfrac {\ pi ^ 2} {6} = \ dfrac {1} {1 ^ 2} + \ dfrac {1} {2 ^ 2} + \ dfrac {1 } {3 ^ 2} + \ dfrac {1} {4 ^ 2} + \ dfrac {1} {5 ^ 2} + \ dfrac {1} {6 ^ 2} … \ tag * {}

Identidade real de Euler.

Citações:

Meu professor: Tan Nguyen

Resposta

Oh cara, você estragou tudo! Não é assim que você faz a pergunta, venha em !

Você faz a seguinte pergunta: O que é

\ displaystyle \ lim\_ {n \ to \ infty} e ^ {- n} \ sum\_ {k = 0} ^ n \ frac {n ^ k} {k!} \, \, \ text {?}

E então você senta e aprecia o espetáculo de todos dizendo que é obviamente 1, e eles explicam o porquê, e é tão claro que não nem mesmo exijo uma prova, mas você pede a eles para provar de qualquer maneira, e eles tentam, e eles falham (ou pior: sucesso), e você pergunta se eles ainda pensam que é 1, e eles dizem que sim, mas parte da confiança se foi e você os joga pelo tempo que quiser, até informá-los que estão exatamente 100\% de desconto.

Então, por que todo mundo acha que esse limite é igual a 1 e por que isso não é verdade e por que é realmente \ frac {1} {2}?

Bem, para n muito grande, a soma \ sum\_ {k = 0} ^ n \ frac {n ^ k} {k!} é sobre e ^ n. Certo? É apenas a série de Taylor da função exponencial. Então, nós multiplicamos isso por e ^ {- n} e obtemos apenas cerca de 1, e conforme deixamos n crescer, isso se torna cada vez mais preciso, então o limite tem que ser simplesmente 1. Eu quero dizer tem que.

Certo?

Errado.

Então, o que há de errado aqui? Bem, você pode se sentir um pouco desconfortável com esse negócio de Taylor. Quero dizer, claro, a soma

\ displaystyle \ sum\_ {k = 0} ^ n \ frac {x ^ k} {k!}

É uma soma parcial do Taylor série de e ^ x, então seu limite para x fixo como n \ a \ infty é de fato e ^ x. Mas aqui estamos fazendo algo um pouco suspeito: estamos pedindo a n para desempenhar uma função dupla como intervalo de soma e variável da série de potência.

Uma coisa, de qualquer forma, deve estar clara: a declaração

\ displaystyle e ^ n =? = \ lim\_ {n \ to \ infty} \ sum\_ {k = 0} ^ {n} \ frac {n ^ k} {k!}

não faz sentido algum. A variável n está livre do lado esquerdo e limitada à direita.

Ok. Portanto, essa interpretação ingênua foi jogada pela janela. Como avaliamos esse limite?

Há uma maneira bonita e magistral de resolver isso que é quase um truque de mão. É assim: esta é precisamente a probabilidade limite de uma variável de Poisson aleatória com o parâmetro \ lambda = n ser menor que sua expectativa. Essa variável é distribuída exatamente como a soma de n variáveis ​​Poisson independentes com o parâmetro \ lambda = 1, e tal soma (normalizada por sua variância, \ sqrt {n}, o que não importa aqui) converge na distribuição para um normal distribuição. Qual é a probabilidade de que uma variável aleatória normal seja menor do que sua média? Por que é \ frac {1} {2}, é claro. Feito. QED.

Espere, o quê?

Sim, realmente. Se você conhece o Teorema do Limite Central, isso é exatamente o que diz se você pegar as variáveis ​​aleatórias X\_1, X\_2, \ ldots, cada uma das quais é Poisson (1). Bom exercício de rotina para aplicar CLT a variáveis ​​aleatórias de Poisson.

Mas e se você não souber sobre a CLT ou simplesmente não lhe ocorrer interpretar esse limite como uma probabilidade?

Então, honestamente, isso é bastante problema difícil. O CLT é um teorema poderoso que esconde um pouco da teoria, oferecendo-o virtualmente de graça. Sem isso, você está por conta própria aqui e não conheço uma maneira realmente fácil de provar isso. Algumas manipulações e transformações integrais inteligentes são necessárias.

Deixe uma resposta

O seu endereço de email não será publicado. Campos obrigatórios marcados com *